Find the value of X!!

Find The Value Of X!!

Answers

Answer 1

The angle made by one chord and tangent of the circle is 32.5 degrees.

What is the Alternate Segment Theorem?

The Alternate Segment Theorem is a theorem in geometry that relates the angles formed by a line that is tangent to a circle and a chord of that circle. The theorem states that the angle formed by a tangent and a chord of a circle is equal to the angle that is subtended by the chord in the opposite segment of the circle

In a circle, the angle formed by a chord and a tangent that intersect at a point on the circle is equal to half the measure of the arc intercepted by the chord.

Therefore, if the arc intercepted by the chord is 65 degrees, then the angle formed by the chord and the tangent is half of 65 degrees, which is:

65 degrees / 2 = 32.5 degrees

So, the angle X made by the chord and the tangent is 32.5 degrees.

Therefore, the angle made by one chord and tangent of the circle is 32.5 degrees.

To learn more about the alternate angle visit:

https://brainly.com/question/20876495

#SPJ1

Answer 2

[tex]x^{0}[/tex] has a value of [tex]27.5[/tex] degrees.

What are types and value?

Values are the benchmarks or ideals by which we judge the acts, traits, possessions, or circumstances of others. Values that are embraced by many include those of beauty, honesty, fairness, harmony, and charity. When considering values, it might be helpful to categorise them into one of three categories: Personal values are those that an individual upholds.

What are the two major categories of value?

Values come in two varieties. They serve as either terminal or auxiliary values for Rokeach. Terminal values always are end-states whereas qualities are always forms of conduct. Individuals think that acting in line with cognitive factors and reaching terminal values are always related.

We find the value of [tex]x^{0}[/tex]

[tex]Angle P = 1/2 (mAC-AB)[/tex]

[tex]x^{0}=\frac{1}{2} (120^{0}- 65^{0} )[/tex]

[tex]x^{0} =\frac{1}{2}*55^{0}[/tex]

Therefore, [tex]x^{0}= 27.5^{0}[/tex]

To know more about value visit:

https://brainly.com/question/30760879

#SPJ1


Related Questions

Find the value of each variable.

Answers

The values of the variables in the semicircle shown are:

x = 63 degrees; y = 90 degrees.

What is the Angle Inscribed in a Semicircle Theorem?

A semi-circle is exactly half of a full circle and has a measurement of 180 degrees; the two endpoints of the diameter form the endpoints of the semi-circle. If an angle is enclosed inside a semi-circle, the angle formed measures 90 degrees.

Therefore, it means the value of the variable, y = 90 degrees.

Thus, using the triangle sum theorem, we have:

x = 180 - 90 - 27

x = 63 degrees.

Learn more about angle inscribed in a semicircle on:

https://brainly.com/question/3538263

#SPJ1

Given f(x)=5-3x, if f(x)=-19, find x.

Answers

f(x)=5-3x
-19=5-3x
-24=-3x
24=3x
x=24/3
x=8

5.7. Suppose n = 2911 and e = 11. Encrypt the following messages as in
Example (5.3).
a) "OK"
b) "HELP" (Break this up into two blocks.)

Answers

Note that,
the encrypted message for "OK" is the pair of numbers (616, 2385).
and the final encrypted message for "HELP" is the sequence of numbers (738, 1277, 1479, 2252).

How do you Encrypted a Message?

To encrypt a message using RSA, we need to first represent the message as numbers using a suitable encoding scheme. For simplicity, we can use the ASCII code for each character, which is a standard encoding scheme for text.

a) To encrypt "OK", we first convert each letter to its corresponding ASCII code:

"O" = 79

"K" = 75

Next, we use the RSA encryption formula:

C ≡ [tex]M^{e}[/tex] (mod n)

For "O", we have C ≡ 79¹¹ (mod 2911) ≡ 616 (mod 2911)

For "K", we have C ≡ 75¹¹ (mod 2911) ≡ 2385 (mod 2911)

Therefore, the encrypted message for "OK" is the pair of numbers (616, 2385).

b) To encrypt "HELP", we break it up into two blocks:

Block 1: "HE"

Block 2: "LP"

For block 1, we have:

"H" = 72

"E" = 69

Using the RSA encryption formula, we get:

C1 ≡ 72¹¹ (mod 2911) ≡ 738 (mod 2911)

C2 ≡ 69¹¹ (mod 2911) ≡ 1277 (mod 2911)

Therefore, the encrypted message for "HE" is the pair of numbers (738, 1277).

For block 2, we have:

"L" = 76

"P" = 80

Using the RSA encryption formula, we get:

C3 ≡ 76¹¹ (mod 2911) ≡ 1479 (mod 2911)

C4 ≡ 80¹¹ (mod 2911) ≡ 2252 (mod 2911)

Therefore, the encrypted message for "LP" is the pair of numbers (1479, 2252).

The final encrypted message for "HELP" is the sequence of numbers (738, 1277, 1479, 2252).

Learn more about Encrypted Message on:

https://brainly.com/question/30408255

#SPJ1

Are the fractions 2/2 and 8/8 equivalent fractions

Answers

Answer:

Step-by-step explanation:

yes since they are both divisible by their denominators and equal the same thing

Answer:

yes, they are equivalent

Step-by-step explanation:

2/2 = (2/2)x(4/4) = 8/8 = 1

Evan is going to invest in an account paying an interest rate of 5.4% compounded annually. How much would Evan need to invest, to the nearest dollar, for the value of the account to reach $1,360 in 5 years

Answers

On solving the provided question, we can say that - the value of the simple interest will be $ 881.28.

What is interest ?

Multiplying the principal by the interest rate, time, and other factors yields simple interest. Simple return equals principle times interest times hours is the marketed formula. It is easiest to compute interest using this formula. A percentage of the principle balance is how interest is most commonly computed. The interest rate on the loan is known as this percentage.

here,

we have

P = 1360;

R = 5.4 ;

T = 12

so, we get,

SI = 1360 X 5.4 X 12 /100

SI =88128/100

= 881.28

Hence, On solving the provided question, we can say that - the value of the simple interest will be $ 881.28.

To know more about simple interest visit:

https://brainly.com/question/25845758

#SPJ9

Find the value of X. Round to the nearest tenth

Answers

Answer:i don't know the answer

Step-by-step explanation: i don't konw

A container contains 145.2 ounces of lemonade. If the lemonade is poured equally into 15 cups, how many ounces will be poured into each cup?

A. 8.78
B. 9.12
C. 9.64
D. 9.68

Show answer.

Answers

Answer: D. 9.68

Step-by-step explanation:

145.2 oz/ 15 cups = 9.68 oz per cup

145.2/15 will then equal to (D) 9.68


Please help need to get a good score

Answers

I provided a picture of it helps you if it does could I get Brainliest please

A normal population has a mean of $76 and standard deviation of $6. You select random samples of 40.

1. What is the probability that a sample mean is less than $75? (Round z-value to 2 decimal places and final answer to 4 decimal places.)

2. What is the probability that a sample mean is between $75 and $77? (Round z-value to 2 decimal places and final answer to 4 decimal places.)

3. What is the probability that a sample mean is between $77 and $78? (Round z-value to 2 decimal places and final answer to 4 decimal places.)

4. What is the probability that the sampling error ( x¯

− μ) would be $1.50 or less? (Round z-value to 2 decimal places and final answer to 4 decimal places.)

Answers

Using a z-table, the probability of a z-score less than 1.58 is 0.9429 (rounded to 4 decimal places).

What is probability?

Probability is a measure of the likelihood or chance of an event occurring. It is expressed as a number between 0 and 1, where 0 represents an impossible event and 1 represents a certain event. The probability of an event can be calculated by dividing the number of favorable outcomes by the total number of possible outcomes. It is used in various fields such as mathematics, statistics, science, and finance to make predictions and analyze data.

Here,

1. The z-score for a sample mean of $75 is calculated as:

z = (75 - 76) / (6 / √(40)) = -2.36

Using a z-table, the probability of a z-score less than -2.36 is 0.0099 (rounded to 4 decimal places).

2. The z-score for a sample mean of $75 is calculated as:

z1 = (75 - 76) / (6 / √(40))

= -2.36

The z-score for a sample mean of $77 is calculated as:

z2 = (77 - 76) / (6 / √(40))

= 0.79

Using a z-table, the probability of a z-score between -2.36 and 0.79 is 0.8669 (rounded to 4 decimal places).

3. The z-score for a sample mean of $77 is calculated as:

z1 = (77 - 76) / (6 / √(40))

= 0.79

The z-score for a sample mean of $78 is calculated as:

z2 = (78 - 76) / (6 / √(40))

= 1.57

Using a z-table, the probability of a z-score between 0.79 and 1.57 is 0.0823 (rounded to 4 decimal places).

4. The standard error of the mean (SEM) is calculated as:

SEM = standard deviation / sqrt(sample size)

SEM = 6 / √(40) = 0.9487

The z-score for a sampling error of $1.50 is calculated as:

z = 1.50 / 0.9487 = 1.58

Using a z-table, the probability of a z-score less than 1.58 is 0.9429 (rounded to 4 decimal places).

To know more about probability,

https://brainly.com/question/30034780

#SPJ1

4. Given that cos theta= 3/10 and 3pi/2 < theta < 2pi, find the exact value of each of the following:

a) sin 2theta

b) The quadrant in which the angle theta/2 is located.

B) cos theta/2

Answers

(a) The value of sin 2θ is -3√(91)/50.

(b)  θ/2 is located in the third quadrant.

(c) The value of cos θ/2 is -√65/10.

What is the value of the sine and cosine functions?

We know that cos θ = 3/10, so we can find sin θ using the Pythagorean identity:

sin² θ + cos² θ = 1

sin²θ + (3/10)² = 1

sin²θ = 1 - (3/10)² = 91/100

sin θ = ±√(91)/10

Since 3π/2 < θ < 2π,

we know that sin θ < 0, so sin θ = -√(91)/10.

a) To find sin 2θ, we can use the double angle formula:

sin 2θ = 2 sin θ cos θ

sin 2θ = 2 (-√(91)/10) (3/10)

sin 2θ = -3√(91)/50

b) To find the quadrant in which θ/2 is located, we first need to find θ/2:

θ/2 = (3π/2 + 2π)/2 = 5π/4

5π/4 is in the third quadrant, so θ/2 is located in the third quadrant.

c) To find cos θ/2, we can use the half angle formula:

cos θ/2 = ±√((1 + cos θ)/2)

Since 3π/2 < θ < 2π, we know that cos θ < 0, so we take the negative square root:

cos θ/2 = -√((1 + 3/10)/2)

cos θ/2 = -√(13/20)

Simplifying the radical by dividing both numerator and denominator by 4:

cos θ/2 = -√(13)/2√5

Multiplying numerator and denominator by √5 to rationalize the denominator:

cos θ/2 = -√65/10

Learn more about quadrant here: https://brainly.com/question/25038683

#SPJ1

Help ASAP DUE IN 30 MINUTES ​

Answers

Answer:

53 in2 is the answer for this question

Answer:

53

Step-by-step explanation:

If you divide the figure into two parts by extending the 4 in side, you get a right triangle and a rectangle.

Area of rectangle:

6*8 = 48 in²

Area of triangle:

1/2*(13 - 8)*(6 - 4) = 1/2 times 5 times 2 = 5 in²

Total area is:

48 + 5 = 53 in²

hope this helps x

Based on the family the graph below belongs to, which equation could represent the graph?

On a coordinate plane, a curve starts at (0, 2) and curves up and to the right in quadrant 1.
y = 2 Superscript x Baseline 3
y = log (2 x) + 3
y = 2 x squared + 2
y = StartFraction 1 Over 2 x EndFraction + 2

Answers

Answer:

Second option [tex]y=\text{log}(2\text{x})+3[/tex]

Solution:

Based on the family of graphs shown in the attached file, the equation could represent the graph is [tex]y=\text{log}(2\text{x})+3[/tex]

This graph is the graph of the function [tex]y=\text{log}(\text{x})[/tex] stretched horizontally by a factor of 2 and translated 3 units upward.

Answer:

B

Step-by-step explanation:

Edge 2023

what is a prime number ​

Answers

Answer: A prime number is a positive integer greater than 1 that has no positive integer divisors other than 1 and itself. In other words, a prime number is a number that is only divisible by 1 and itself.

Step-by-step explanation:

For example, 2, 3, 5, 7, 11, 13, 17, 19, 23, 29, 31, and so on, are prime numbers because they can only be divided by 1 and themselves without any remainder.

However, 4 is not a prime number because it can be divided by 1, 2, and 4, and 6 is not a prime number because it can be divided by 1, 2, 3, and 6.

Answer:

A prime number is a number that can be multiplied by one and itself~
eg- 2,3,5,7,11
Let me know if this helps

Bug S Bug S and Bug F is fast. Both bugs start at 0 on a number line and move in the positive direction. The bugs leave 0 at the same time and move at constant speeds. Four seconds later, F is at 12 and S is at 8. When will F and S be 100 units apart?

Answers

Answer:

Let's call the speed of Bug F v_F and the speed of Bug S v_S. Since both bugs started at 0, we can express their positions at any time t as:

Position of Bug F = 12 + v_F * t

Position of Bug S = 8 + v_S * t

To find out when F and S will be 100 units apart, we need to find the time t at which their positions differ by 100 units. In other words, we need to solve the following equation:

|12 + v_F * t - (8 + v_S * t)| = 100

We can simplify this equation by expanding the absolute value and rearranging the terms:

|4 + (v_F - v_S) * t| = 100

Now we can split this equation into two cases:

Case 1: 4 + (v_F - v_S) * t = 100

In this case, we have:

v_F - v_S > 0 (since Bug F is faster)

t = (100 - 4) / (v_F - v_S)

Case 2: 4 + (v_F - v_S) * t = -100

In this case, we have:

v_F - v_S < 0 (since Bug S is faster)

t = (-100 - 4) / (v_F - v_S)

Since we're only interested in positive values of t, we can discard the second case. Therefore, the time at which F and S will be 100 units apart is:

t = (100 - 4) / (v_F - v_S)

t = 96 / (v_F - v_S)

We don't know the values of v_F and v_S, but we can use the fact that Bug F is at 12 and Bug S is at 8, four seconds after they started. This gives us two equations:

12 = 4v_F + 0v_S

8 = 4v_S + 0v_F

Solving these equations for v_F and v_S, we get:

v_F = 3

v_S = 2

Substituting these values into the equation for t, we get:

t = 96 / (3 - 2)

t = 96

Therefore, F and S will be 100 units apart 96 seconds after they start.

3) Given that f(x) = 3x – 5 g(x) = 2x – 6 and h(x) = x + 4
4 2x
Find:- i) f(-3) = ii) g[f(0)] = iii) f[h(2)] =

iv) hᴏf(x) v) h-1(1) =

Answers

The Answer for the given functions are:

i) f(-3) = -14.

ii) g[f(0)] = -16.

iii) f[h(2)] = 13.

iv)  hᴏf(x) = 3x - 1.

v)  h-1(1) = -3.

What is the functiοn nοtatiοn?

Functiοn nοtatiοn is a way οf representing a functiοn using algebraic symbοls. It is a shοrthand way οf expressing a relatiοnship between twο quantities οr variables, where οne variable depends οn the οther.

i) Tο find f(-3), we substitute x = -3 in the expressiοn fοr f(x) and simplify:

f(-3) = 3(-3) - 5 = -9 - 5 = -14

Therefοre, f(-3) = -14.

ii) Tο find g[f(0)], we first evaluate f(0) and then substitute that value intο g(x):

f(0) = 3(0) - 5 = -5

g[f(0)] = g(-5) = 2(-5) - 6 = -10 - 6 = -16

Therefοre, g[f(0)] = -16.

iii) Tο find f[h(2)], we first evaluate h(2) and then substitute that value intο f(x):

h(2) = 2 + 4 = 6

f[h(2)] = f(6) = 3(6) - 5 = 18 - 5 = 13

Therefοre, f[h(2)] = 13.

iv) Tο find hᴏf(x), we substitute f(x) intο the expressiοn fοr h(x) and simplify:

hᴏf(x) = h[f(x)] = f(x) + 4 = (3x - 5) + 4 = 3x - 1

Therefοre, hᴏf(x) = 3x - 1.

v) Tο find h-1(1), we need tο sοlve fοr x in the equatiοn h(x) = 1:

h(x) = x + 4 = 1

Subtracting 4 frοm bοth sides, we get:

x = 1 - 4 = -3

Therefοre, h-1(1) = -3.

To learn more about function notation, Visit

https://brainly.com/question/27915724

#SPJ1

The box plots show the weights, in pounds, of the dogs in two different animal shelters.

Weights of Dogs in Shelter A
2 box plots. The number line goes from 6 to 30. For the weights of dogs in shelter A, the whiskers range from 8 to 30, and the box ranges from 17 to 28. A line divides the box at 21. For shelter B, the whiskers range from 10 to 28, and the box ranges from 16 to 20. A line divides the box at 18.
Weights of Dogs in Shelter B

Which animal shelter has the dog that weighs the least?
shelter A

Answers

Step-by-step explanation:

The minimum weight for shelter A is not provided in the given information, but we can compare the minimum weight of shelter B with shelter A's box plot.

As per the given information, the whisker of shelter A ranges from 8 to 30, which means the minimum weight in shelter A is 8 pounds. On the other hand, the whisker of shelter B ranges from 10 to 28, which means the minimum weight in shelter B is 10 pounds. Therefore, shelter A has the dog that weighs the least.

Answer:

Your answer is correct, it's shelter A.

Step-by-step explanation:

A quality assurance check is 91% accurate for non-defective devices and 97% accurate for defective devices. Of the devices checked, 84% are not defective. What is the probability of an incorrect conclusion? Round your answer to the nearest tenth of a percent.

Answers

Answer: To solve the problem, we can use Bayes' theorem. Let D be the event that a device is defective, and let A be the event that the quality assurance check concludes that a device is defective.

We want to find P(A and not D) + P(not A and D), which represents the probability of an incorrect conclusion.

We know that P(D) = 1 - P(not D) = 1 - 0.84 = 0.16, and that P(A | not D) = 0.03 and P(A | D) = 0.97.

Using Bayes' theorem, we can compute:

P(not A | not D) = 1 - P(A | not D) = 1 - 0.03 = 0.97

P(not A | D) = 1 - P(A | D) = 1 - 0.97 = 0.03

Therefore,

P(A and not D) = P(not D) * P(A | not D) = 0.84 * 0.03 = 0.0252

P(not A and D) = P(D) * P(not A | D) = 0.16 * 0.03 = 0.0048

So the probability of an incorrect conclusion is:

P(A and not D) + P(not A and D) = 0.0252 + 0.0048 = 0.03

Therefore, the probability of an incorrect conclusion is 0.03, or 3% (rounded to the nearest tenth of a percent).

Why was this answer deleted prior?

Suppose you have income of $24, the price of x is $2, the price of y is $4. Your utility is given by the function U=3x^2/3y^1/3. Solve for utiltiy maximizing bundle. Suppose the government intewrvenes in this market and limits purchases of x to no more than 4 units . Are you better off? You need to demonstrate graphically or with calculations

Answers

Answer:

Step-by-step explanation:

To find the utility-maximizing bundle of goods, we need to solve for the values of x and y that maximize U while still satisfying the budget constraint. The budget constraint can be written as:

2x + 4y = 24

or

x + 2y = 12

We can use the method of Lagrange multipliers to solve for the utility-maximizing values of x and y subject to this constraint. The Lagrangian function is:

L = 3x^(2/3)y^(-1/3) + λ(x + 2y - 12)

Taking partial derivatives with respect to x, y, and λ, we get:

dL/dx = 2x^(-1/3)y^(-1/3) + λ = 0

dL/dy = -x^(2/3)y^(-4/3) + 2λ = 0

dL/dλ = x + 2y - 12 = 0

Solving these equations simultaneously, we get:

x = 6

y = 3

So the utility-maximizing bundle is 6 units of x and 3 units of y.

To see if the individual is better off with the government intervention, we can plot the budget line and the indifference curve for the utility-maximizing bundle with and without the limit on x.

Without the limit, the budget line is the same as before (x + 2y = 12), and the indifference curve for the utility-maximizing bundle passes through the point (6, 3) on the graph.

With the limit, the budget line becomes x = 4, since the individual is prohibited from purchasing more than 4 units of x. The corresponding budget line has a slope of -1/2 and intercepts the y-axis at 6.

If we draw the indifference curve for the utility-maximizing bundle of (4,4), which lies on the budget line, we can see that the individual is not better off with the government intervention. This is because the slope of the budget line under the intervention is steeper, so the individual would have to give up more y than x to afford the same amount of utility. Thus, the individual would have to move to a lower indifference curve with lower utility.

Therefore, the individual is not better off with the government intervention.

3 Cassie wants to determine the length of the shadow that a 60-foot tall telephone pole casts without measuring it. If Cassie's mailbox, which is 42 inches in height, casts a shadow that is 31.5 inches in length, how long is the shadow that the telephone pole casts? A. 43 feet B. 45 feet C. 52 feet D. 55 feet ​

Answers

The answer is (B) 45 feet

Step-by-step explanation:

We can use proportions to solve this problem.

Let x be the length of the shadow cast by the telephone pole. Then we have:

(42 / 31.5) = (60 / x)

We can cross-multiply to get:

42x = 31.5 * 60

Simplifying this equation, we get:

x = (31.5 * 60) / 42

x = 45 feet

Therefore, the length of the shadow that the telephone pole casts is 45 feet.

What will be the result of substituting 2 for x in both expressions below?

Answers

Substituting for x in an expression means replacing the variable x with a specific value or expression. This is often done to evaluate the expression for that particular value or to simplify the expression.

What is the substituting for x in expressions?

Substituting 2 for x in the first expression, we get:

[tex]1/2(2) + 4(2) + 6 - 1/2(2) - 2 = 1 + 8 + 6 - 1 - 2 = 12[/tex]

Substituting 2 for x in the second expression, we get:

[tex]2(2) + 2 - 1 = 5[/tex]

One expression equals 5 when substituting 2 for x, and the other equals 2 because the expressions are not equivalent.

Therefore, the first expression evaluated with x = 2 is 12, and the second expression evaluated with x = 2 is 5. Since they do not have the same value, the correct option is:

Learn more about expressions here:

https://brainly.com/question/14083225

#SPJ1

The given question is incomplete. The complete question is given below:

What will be the result of substituting 2 for x in both expressions below? One-half x + 4 x + 6 minus one-half x minus 2 Both expressions equal 5 when substituting 2 for x because the expressions are equivalent. Both expressions equal 6 when substituting 2 for x because the expressions are equivalent. One expression equals 5 when substituting 2 for x, and the other equals 2 because the expressions are not equivalent. One expression equals 6 when substituting 2 for x, and the other equals 2 because the expressions are not equivalent.

Write an expression describing all the angles that are coterminal with 8°. (Please use the variable k in your answer. Give your answer in degrees, but do not include a degree symbol in your answer.)

Answers

the expression describing all the angles that are coterminal with 8° is: θ = 8° + 360°k, where k is an integer.

How to solve and what is angle?

An angle of 8° has an initial side on the positive x-axis and rotates counterclockwise by 8°.

Any angle coterminal with 8° can be expressed as:

θ = 8° + 360°k

where k is an integer.

Therefore, the expression describing all the angles that are coterminal with 8° is:

θ = 8° + 360°k, where k is an integer.

An angle is a geometric figure formed by two rays, called the sides of the angle, that share a common endpoint, called the vertex of the angle. Angles are typically measured in degrees or radians and are used to measure the amount of rotation or turn between two intersecting lines or planes.

To know more about angle related question visit:

https://brainly.com/question/28451077

#SPJ1

How to get the unadjusted cost of sales in cost and management accounting

Answers

Answer:

Step-by-step explanation:

To calculate unadjusted cost of goods sold, sum the beginning inventory value and the cost of goods manufactured, then subtract the ending inventory value.

Twelve friends share 4 cookies equally. What fraction of a cookie does each friend get? Write in simpliest form

Answers

Answer:

2/5 of the cookie

Step-by-step explanation:

12 friends need to split 4 cookies

4 cookies needs to divided by 10 people

[tex]\frac{4cookies}{10 people}[/tex] = [tex]\frac{4}{10}[/tex]

simplify: [tex]\frac{4}{10} = \frac{2}{5}[/tex]

What’s 4559.886 rounded to the nearest inch

Answers

Answer:

Step-by-step explanation:

Assuming that the original measurement is in millimeters, since they are commonly used in precision applications, we know that 1 millimeter is equal to 0.03937 inches. Therefore, we can convert 4559.886 millimeters to inches by multiplying by 0.03937:

4559.886 mm * 0.03937 in/mm = 179.72467632 in (rounded to 8 decimal places)

To round this to the nearest inch, we need to look at the first decimal place after the decimal point. In this case, the digit in the first decimal place is 7, which is greater than or equal to 5. Therefore, we round up to the nearest inch, which gives us:

179.72467632 in rounded to the nearest inch is 180 in.

Therefore, 4559.886 rounded to the nearest inch is 180.

I need help with my homework

Answers

To find the length of a line segment in a circle, use the formula [tex]d = 2r[/tex] [tex]sin(t/2)[/tex] , where r is the radius of the circle and t is the angle between the radii. The length of segment DE is  [tex]5[/tex] units.

What is the formula for circle segment length?

We can use the similar triangles property to find the missing length of segment DE in the given figure. Because triangles ABD and CBE are similar, we can use a proportion to find the length of DE:

[tex]CB/BE = AB/BD[/tex]

With the given values, we get:

[tex]3/6 = 5/(5 + DE)[/tex]

When we simplify and solve for DE, we get:

[tex]3(5 + DE) = 6 * 5 \s15 + 3DE = 30[/tex]

[tex]3DE = 15 \sDE = 5[/tex]

Therefore, segment DE has a length of 5 units.

Learn more about circle here:

https://brainly.com/question/28612561

#SPJ1

Find the sum. -
3/4 + 1/2 =

Answers

Answer:

Is the problem -3/4 + 1/2 or 3/4 + 1/2?

I'll just do both then.

-3/4 + 1/2 = -1/4

3/4 + 1/2 = 5/4 or 1 1/4

Step-by-step explanation:

You're welcome.

Answer:

[tex] \frac{3}{4 } + \frac{1}{2} [/tex]

take lcm of denominators i.e. 4and 2

so, the lcm of 4 and 2 is 4.

[tex] \frac{3}{4} + \frac{2}{4} [/tex]

[tex] \frac{3 + 2}{4} [/tex]

[tex] \frac{5}{4} [/tex]

Step-by-step explanation:

hope this will be helpful:)

a restaurant menu offers tomato, broccoli, and potato soups. customers order potato soup 50% of the time, broccoli 30% of the time, and tomato 20% of the time the chef designs a simulation to estimate how many of her next 10 customers will order broccoli soup. she labels five index cards p, three cards b, and two cards t. she shuffles the cards and randomly chooses a card from the pile. she records the letter, returns the card, and draws another. she repeats this process for a total of 10 draws. she completes this simulation five times. based on the simulations, how accurate is the chef's estimation regarding broccoli soup orders?

Answers

The simulation suggests that the chef's estimation of 30% broccoli soup orders is reasonable, but actual orders will vary from one set of customers to the next.

The chef's estimation of 30% broccoli soup orders seems reasonable based on the probabilities given, but the actual number of broccoli soup orders will likely vary from one set of 10 customers to the next. To estimate the accuracy of the chef's estimation, she conducts a simulation by shuffling cards representing the soup choices and randomly selecting one for each of the 10 customers. She repeats this process five times.

Based on the simulation results, we can see that the actual number of broccoli soup orders varies quite a bit from the expected value of 1.5. However, this is to be expected due to the random nature of the simulation.

To further estimate the accuracy of the chef's estimation, we could calculate the mean and standard deviation of the results to get a better sense of the distribution of possible outcomes.

Overall, the simulation suggests that the chef's estimation of 30% broccoli soup orders is a reasonable estimate, but the actual number of broccoli soup orders will likely vary from one set of 10 customers to the next.

To learn more about estimation please click on below link        

https://brainly.com/question/30982574

#SPJ1

Complete the ratio table to convert the units of time from hours to weeks or weeks to hours.
Hours:
168 1 week
1,008. ____week
_____. 5 weeks

Answers

Answer:

6 weeks and 840 hours

Step-by-step explanation:

There are 168 hours in one week.

24 hrs/day * 7 days = 168 hours

1008 hours ÷ 24 hours(1 day) = 42 days ÷ 7 days in a week = 6 weeks

168 hours/week * 5 weeks = 840 hours

PLS COMPLETE ALL OF IT!! 50 POINTS!​

Answers

A. The length of the cord needed to reach corner C is 17.6 m

B. The distance between the electrical outlet and corner N is 14.3 m

A. How do i determine the length of cord needed?

The length of the cord needed can be obtained as follow:

Length BC = Opposite = 8 mAngle (θ) = 27°Length of cord =?

Sine θ = opposite / hypotenuse

Sine 27 = 8 / Length of cord

Cross multiply

Length of cord × sine 27 = 8

Divide both sides by sine 27

Length of cord = 8 / sine 27

Length of cord = 17.6 m

B. How do i determine the distance between electrical outlet and corner N

First, we shall determine the length OB. Details below:

Angle (θ) = 27°Length BC = Opposite = 8 mLength OB =?

Tan θ = Opposite / Adjacent

Tan 27 = 8 / Length OB

Cross multiply

Length OB × tan 27 = 8

Divide both sides by tan 27

Length OB = 8 / tan 27

Length OB = 15.7 m

Finally, we shall determine the distance between the electrical outlet and corner N. Details below:

Length OB = 15.7 mLength BN = 30 mLength ON = Distance =?

Length BN = Length OB + Length ON

30 = 15.7 + Distance

Collect like terms

Distance = 30 - 15.7

Distance = 14.3 m

Learn more about missing varables:

https://brainly.com/question/27481210

#SPJ1

a bakery has 17 pounds of flour they want to put into 6 containers. They put the same amount of flour in each container. If they want to use up all the flour, how much flour should they put in each container?

Answers

Answer:

2.83333 pounds of flour in each container

Other Questions
Help I will mark you as a brainliest What could the shattered glass paperweight symbolize here?Select one:a. Mr. Charringtons broken dreams.b. Big Brothers shattered control over Winston.c. The destruction of Winstons private world.d. The falling apart of Winstons family. what's the field strength on the loop axis at 10.0 cm from the loop center? express your answer in microtesla. Tanya painted a mural that was 8 feet tall. The area of the mural was 224 square feet. What is the length of Tanyas mural? Determine whether the underlined word is an adverb, and identify the type of word it modifies. Select "Check Answers" to see how well you did. 1. The Irish Festival is extremely popular. if i deposit $5,250 into an account today that pays 8.3% interest per annum (i.e., p.a.), how much will be in my account exactly two years from today? question 5 options: $4,476.13 $6,668.75 $5,322.88 $6,157.67 $5,685.75 If the graph of a polynomial function P(x) has -intercepts at x = - 4, x = 0, x * 1 point= 5, which of the following must be true for P(x)? (x + 5) is a factor of the polynomial. (x-4) is a factor of the polynomial.' The degree of the polynomial is 3. The degree of the polynomial is greater than or equal to 3. Using the information on this page, explain in your own words at least three ways in which the U.S. and Canadian economies are interrelated.Respond in at least three complete sentences. why was it not until the late eighteenth century that large numbers of individuals considered slavery to be morally wrong? solve ABC subject to the given conditions if possible. Round the lengths of the sides and measures of the angles (in degrees) to one decimal place it necessary.B=64 degrees, a=25, b=41 In a car park,the number of cars: the number of vans = 7:4the number of vans: the number of lorries = 3:2The total number of cars, vans and lorries in the car park is 205How many vans are in the car park? when a salesperson has been introduced to a sales lead by another customer, that salesperson has been provided a(n) . angelica vaughn owns a home in edmond, oklahoma. she is worried about a tornado causing damage to her home. the tornado is an example of a: which one of the following amino acids r groups (a.k.a. side chain) is most likely to participate in hydrogen bonding with water? group of answer choices asparagine alanine leucine phenylalanine valine In one to two sentences, describe a theme of the excerpt. Self porptrait What is the difference in volume (in mL) of oxygen produced between the low-light and high-light plants after 45 minutes? jean piaget believed that human behavior is based on specific human needs that must often be met in a specific order true false Kilroy uses the net method to record sales on account. kilroy sells goods on account for $1,000 with terms 2/10, n/30. the journal entry to record this transaction will include (select all that apply.) multiple select question. debit to sales, $1,000.credit to sales, $980.credit to sales, $1,000. credit to accounts receivable, $980. debit to accounts receivable, $1,000.debit to accounts receivable, $980. Find the area of the shaded sector of the circle As a particle moves 12 meters along an electric field of strength of 80 Newtons per Coulomb its electrical potential energy decreases by 5.2 x 10^-18 Joules.What is the particle charge?Giving out brainliest please help this is due today.